LSAT and Law School Admissions Forum

Get expert LSAT preparation and law school admissions advice from PowerScore Test Preparation.

User avatar
 Dave Killoran
PowerScore Staff
  • PowerScore Staff
  • Posts: 5852
  • Joined: Mar 25, 2011
|
#59625
Complete Question Explanation
(The complete setup for this game can be found here: lsat/viewtopic.php?t=15933)

The correct answer choice is (E)

The question stem places G and K in a block:

pt44_o04_g2_q10a.png
Since L and P are also aligned in a block, two of three days must be completely filled by those two blocks:

pt44_o04_g2_q10b.png
Thus, because the only two remaining spaces are on the same day, the two remaining dogs—H and S—must also be in a block, and the key to this question is to determine the days on which each block can be placed. Let’s examine each block in closer detail:

  • The GK block: The only rule applicable to this block is the third rule, which states that if K is placed on Monday then G must be placed on Tuesday. Consequently, we can infer that the GK block cannot be placed on Monday since that placement would violate this rule.

    The LP block: The LP block could be placed on any one of the three days.

    The HS block: The only rule applicable to this block is the fourth rule, which states that if S is placed on Wednesday then H must be placed on Tuesday. Consequently, we can infer that the HS block cannot be placed on Wednesday since that placement would violate this rule.

With the information above, we are ready to attack the answer choices.

Answer choice (A): Although H could be placed on Monday, H does not have to be placed on Monday and so this answer choice is incorrect.

Answer choice (B): Although L could be placed on Monday, L does not have to be placed on Monday and so this answer choice is incorrect.

Answer choice (C): Although K could be placed on Tuesday, K does not have to be placed on Tuesday and so this answer choice is incorrect.

Answer choice (D): This answer choice, like answer choice (E), slips a “not” into the middle of the answer (remember, read carefully!). Because P could be placed on Wednesday, this answer choice is incorrect.

Answer choice (E): This is the correct answer choice. As discussed above, the HS block cannot be placed on Wednesday because doing so would violate the fourth rule.
You do not have the required permissions to view the files attached to this post.
 T.B.Justin
  • Posts: 194
  • Joined: Jun 01, 2018
|
#60818
I came to answer choice A:

I had two templates for what happens if H is not on Tuesday, which shows the GK block on Wednesday with H on Monday or on Tuesday with the HS block on Monday; in both H must be Monday, however I did not account that the LP block could be placed in this scenario on Monday which would put the HS block on Tuesday and GK block on Wednesday.
 Brook Miscoski
PowerScore Staff
  • PowerScore Staff
  • Posts: 418
  • Joined: Sep 13, 2018
|
#62689
Justin--yes, it is correct that when you are using templates it is important to be cognizant of the elements that can be repositioned within the template. It's good to take some time up front to be sure you aren't missing a possibility.

Get the most out of your LSAT Prep Plus subscription.

Analyze and track your performance with our Testing and Analytics Package.